ID Content Options
152018311

 Let \(f:\mathbb{R}\rightarrow \mathbb{R},\,g:\mathbb{R}\rightarrow \mathbb{R}\) be differentiable function such that \((fog)(x)=x\)  . If \(f(x)=2x+cosx+sin^2x\) , then the value of \( \sum_{n=1}^{99}g(1+(2n-1)\pi )\) is


Please choose your answer from the right side options

\(1250\pi\)

\((99)^2\frac{\pi}{2}\)

\((99)^2{\pi}\)

\(2500\pi\)

152018312

If \(f:[1,\infty )\rightarrow [1,\infty ]\) is defined by \(f(x)=\frac{1+\sqrt{1+4log_2\,x}}{1}\) then \( \,\,f^{-1}(3)=\)


Please choose your answer from the right side options

\(0\)

\(1\)

\(64\)

\(\frac{1+\sqrt{5}}{2}\)

152018313

If \(α\) and \(β\) are the greatest divisors of \(n(n^2-1)\) and \(2n(n^2+2)\) respectively for all \(n\,\epsilon\, N\) then \(αβ=\)


Please choose your answer from the right side options

\(18\)

\(36 \)

\(27 \)

\(9\)

152018314

Let \(A=\begin{bmatrix} \frac{1}{6} & \frac{-1}{3} &\frac{-1}{6} \\ \frac{-1}{3}& \frac{2}{3} &\frac{1}{3} \\ \frac{-1}6{} & \frac{1}{3} & \frac{1}{6} \end{bmatrix}\) . If \(A^{2016l}+A^{2017m}+A^{2018n}=\frac{l}{\alpha }A \,\, \) for every \(l,m,n\,\epsilon\, N\) , then the value of \(α\) is


Please choose your answer from the right side options

\(\frac{1}{6}\)

\(\frac{1}{3}\)

\(\frac{1}{2}\)

\(\frac{2}{3}\)

152018315

Let \(l,m,n\,\epsilon\, \mathbb{R}\) and \(A=\begin{bmatrix} 1 & r& r^2 &1 \\ r& r^2 &1 & m\\ r^2& 1 &r & n \end{bmatrix}\) . Then the set of all real values of r for which the rank of A is 3, is


Please choose your answer from the right side options

\((0,\infty)\)

\(R\)

\(R-\left \{ 1 \right \}\)

\(R-\left \{ 0 \right \}\)

152018316

The following system of equations

 \(x+y+z=9 \)

\(2x+5y+7z=52\)

\(x+7y+11z=77\)

has


Please choose your answer from the right side options

no solution 

exactly 2 solutions 

only one solution 

infinitely many solutions

152018317

\(Z\) is a complex number such that \(\left | Z \right |\leq 2\) and \( \,\,-\frac{\pi }{3}\leq \,amp\,\,Z\leq \frac{\pi }{3}\) .The area of the region formed by locus of \(Z\) is


Please choose your answer from the right side options

\(\frac{2\pi}{3}\)

\(\frac{\pi}{3}\)

\(\frac{4\pi}{3}\)

\(\frac{8\pi}{3}\)

152018318

The points on the argand plane is given by \(Z_1=-3+5i,\,Z_2=-1+6i,\,Z_3=-2+8i,\,Z_4=-4+7i\) form a


Please choose your answer from the right side options

parallelogram 

rectangle 

rhombus 

square

152018319

When \(n=8,\,(\sqrt{3}+i)^n+(\sqrt{3}-i)^n=\)


Please choose your answer from the right side options

\(−256 \)

\(−128 \)

\(256i \)

\(128i\)

1520183110

If \(2\,cos\frac{7\pi }{5}\) is one of the value of \(z^\frac{1}{5}\) , then \( z =\)


Please choose your answer from the right side options

\(32+ 32i \)

\(−32 \)

\(−1 \)

\(32\)

1520183111

The set of real values of \(x\) for which the inequality \(\left | x-1 \right |+\left | x+1 \right |< 4\) always holds good is


Please choose your answer from the right side options

\((−2,2) \)

\(\left ( -\infty ,-2 \right )\cup \left ( 2,\infty \right )\)

\(\left ( -\infty ,-1\right ]\cup \left ( 1,\infty \right )\)

\(\left ( -2 ,-1\right )\cup \left ( 1,2\right )\)

1520183112

If the roots of the equation \(x^2+x+a=0\) exceed a , then


Please choose your answer from the right side options

\(a>2\)

\(a<-2\)

\(2<a<3\)

\(-2<a<-1\)

1520183113

If the roots of the equation \(\sqrt{\frac{x}{1-x}}+\sqrt{\frac{1-x}{x}}=\frac{5}{2}\) are p and \(q\left ( p>q \right )\) and the roots of the equation \(\left ( p+q \right )x^4-pqx^2+\frac{p}{q}=0\) are \(α,\, β\,, γ\, , δ\) then \(\left ( \sum \alpha \right )^2-\sum \alpha \beta +\alpha \beta \gamma \delta=\)


Please choose your answer from the right side options

\(0\)

\(\frac{104}{25}\)

\(\frac{25}{4}\)

\(\frac{16}{5}\)

1520183114

 The equation \(x^5-5x^3+5x^2-1=0\) has three equal roots. If \(α\) and \(β\) are the two other roots of this equation, then \(α + β + αβ =\)


Please choose your answer from the right side options

\(−4 \)

\(3 \)

\(−2 \)

\( −5\)

1520183115

If all possible numbers formed by using the digits \( 1,2,3,5,7 \) without repetition and they are arranged in descending order, then the rank of the number \(327\) is


Please choose your answer from the right side options

\(31 \)

\(175 \)

\(149 \)

\(271\)

1520183116

If \(a\) is the number of all even divisors and \(b\) is the number of all odd divisors of the number \(10800\), then \(2a+3b=\)


Please choose your answer from the right side options

\(72 \)

\(132 \)

\(96 \)

\(136\)

1520183117

If the coefficient of \(x^5\) in the expansion of \(\left ( ax^2+\frac{1}{bx} \right )^{13}\) is equal to the coefficient of \(x^{-5}\) in the expansion of \(\left ( ax-\frac{1}{bx^2} \right )^{13}\) , then \(ab =\)


Please choose your answer from the right side options

\(1\)

\(\frac{1}{6}\)

\(\frac{7}{6}\)

\(\frac{4}{2}\)

1520183118

For \(n\,\epsilon \,N\) , in the expansion of \(\left ( \sqrt[4]{x^{-3}}+a\sqrt[4]{x^5} \right )^n\), the sum of all binomial coefficients lies between \(200\) and \(400\) and the term independent of \(x\) is \(448\).Then the value of \(a\) is


Please choose your answer from the right side options

\(1\)

\(2\)

\(\frac{1}{2}\)

\(0\)

1520183119

  If \(\frac{x^4+x^3+2x^2-2x+1}{x^3+x^2}=P(x)+\frac{A}{x}+\frac{B}{x^2}+\frac{C}{x+1}\) , then \(A+B+C=\)


Please choose your answer from the right side options

\(P(0)\)

\(P(2)\)

\(P(3)\)

\(P(4)\)

1520183120

If \(A(n)=sin^n\alpha +cos^n\alpha \) then \(A(1)A(4)+A(2)A(5)=\)


Please choose your answer from the right side options

\(A(1)A(2)+A(4)A(5)\)

\(A(1)A(6)+A(2)A(3)\)

\(A(1)A(3)+A(2)A(6)\)

\(A(1)A(2)+A(3)A(6)\)

1520183121

When \(\frac{sin\,9\theta}{cos\,27\theta}+\frac{sin\,3\theta}{cos\,9\theta}+\frac{sin\,\theta}{cos\,3\theta}=k\,(tan\,27\theta-tan\,\theta)\) is defined, then \(k=\)


Please choose your answer from the right side options

\(\frac{\pi}{2}\)

\(-\frac{1}{2}\)

\(\frac{1}{2}\)

\(\frac{\pi}{4}\)

1520183122

If \(x=\sum_{n=0}^{\infty }cos^{2n}\,\theta,\,\,\,y=\sum_{n=0}^{\infty }sin^{2n}\,\theta,\,\,\,z=\sum_{n=0}^{\infty }cos^{2n}\,\theta sin^{2n}\,\theta\)  and \(0<\theta<\frac{\pi }{2}\) , then


Please choose your answer from the right side options

\(xz+yz=xy+z\)

\(xyz=yz+x\)

\(xy+z=xy+zx\)

\(x+y+z=xyz+z\)

1520183123

Number of solutions of the equation \(sin\,x-sin\,2x+sin\,3x=2\,cos^2\,x-2\,cos\,x\) in \( (0,Ï€)\) is


Please choose your answer from the right side options

\(1\)

\(3\)

\(2\)

\(4\)

1520183124

\(2Tan^{-1}\frac{1}{5}+Sec^{-1}\frac{5\sqrt{2}}{7}+2Tan^{-1}\frac{1}{8}=\)


Please choose your answer from the right side options

\(\frac{\pi}{6}\)

\(\frac{\pi}{4}\)

\(\frac{\pi}{3}\)

\(\frac{\pi}{8}\)

1520183125

If \(cosh\,x=\frac{\sqrt{14}}{3},sinh\,x=cos\,\theta \) and \(\,\,-\pi <\theta<-\frac{\pi}{2}\) , then \(\,\,sin\,\theta=\)


Please choose your answer from the right side options

\(\frac{1}{3}\)

\(\frac{2}{3}\)

\(-\frac{1}{3}\)

\(-\frac{2}{3}\)

1520183126

In \(\Delta ABC \,\,\), if \(a=5\,\,\) and \(tan\frac{A-B}{2}=\frac{1}{4}tan\frac{A+B}{2}\), then \(\,\,\sqrt{a^2-b^2}=\)


Please choose your answer from the right side options

\(2\)

\(3\)

\(4\)

\(5\)

1520183127

In a triangle \(ABC\)  if \(A=2B\) and the sides opposite to the angles \(A,B,C \) are \(α+ 1,α− 1\) and \(\alpha\) respectively then \(α =\)


Please choose your answer from the right side options

\(3\)

\(4\)

\(5\)

\(6\)

1520183128

In \(∆ABC\) , right angled at \(A\) , the circumradius, inradius and radius of the excircle opposite to \(A\) are respectively in the ratio \(2:5:λ\) , then the roots of the equation \(x^2-(\lambda -5)x+(\lambda -6)=0\) are


Please choose your answer from the right side options

\(3,4 \)

\( 5,13 \)

\( 1,3 \)

\(8,13\)

1520183129

Let \(3\bar{i}+\bar{j}-\bar{k}\) be the position vector of a point \(B\) . Let \(A\) be a point on the line which is passing through \(B\) and parallel to the vector \(2\bar{i}-\bar{j}+2\bar{k}\) . If \(\left | \overline{BA} \right |=18\) , then the position vector of \(A\) is


Please choose your answer from the right side options

\(-9\bar{i}+7\bar{j}-13\bar{k}\)

\(-9\bar{i}+3\bar{j}+12\bar{k}\)

\(9\bar{i}-3\bar{j}+2\bar{k}\)

\(3\bar{i}-\bar{j}+7\bar{k}\)

1520183130

The vector that is parallel to the vector \(2\bar{i}-2\bar{j}-4\bar{k}\) and coplanar with the vectors \(\bar{i}+\bar{j}\) and \(\bar{j}+\bar{k}\) is


Please choose your answer from the right side options

\(\bar{i}-\bar{k}\)

\(\bar{i}+\bar{j}-\bar{k}\)

\(\bar{i}-\bar{j}-2\bar{k}\)

\(3\bar{i}+3\bar{j}+6\bar{k}\)

1520183131

A line \(L\) is passing through the point \(A\) whose position vector is \(\bar{i}+2\bar{j}-3\bar{k}\) and parallel to vector \(2\bar{i}+\bar{j}+2\bar{k}\) . A plane \(Ï€\) is passing through the points \(\bar{i}+\bar{j}+\bar{k},\bar{i}-\bar{j}-\bar{k}\)  and parallel to the vector \(\bar{i}-2\bar{j}\) . Then the point where this plane \(Ï€\) meets the line \(L\) is


Please choose your answer from the right side options

\(\frac{1}{3}\left ( -7\bar{i}+\bar{j}-19\bar{k} \right )\)

\( -7\bar{i}+\bar{j}-19\bar{k} \)

\( 3\bar{i}+3\bar{j}-\bar{k} \)

\(​​ 2\bar{i}-\bar{j}+\bar{k}\)

1520183132

If the position vector of three points \(A,B,C\) respectively are \(\bar{i}+2\bar{j}+\bar{k}, 2\bar{i}-\bar{j}+2\bar{k}\) and \(\bar{i}+\bar{j}+2\bar{k}\) , then the perpendicular distance of the point \(C\)from the line \(AB\) is,


Please choose your answer from the right side options

\( \sqrt{\frac{3}{11}}\)

\( \sqrt{\frac{4}{11}}\)

\( \sqrt{\frac{6}{11}}\)

\( \sqrt{\frac{8}{11}}\)

1520183133

The volume of a tetrahedron whose vertices are \(4\bar{i}+5\bar{j}+\bar{k},-\bar{j}+\bar{k},3\bar{i}+9\bar{j}+4\bar{k}\) and \(-2\bar{i}+4\bar{j}+4\bar{k}\) is (in cubic units)


Please choose your answer from the right side options

\(\frac{14}{3}\)

\(5\)

\(6\)

\(30\)

1520183134

If the vectors \(\bar{b},\bar{c},\bar{d}\) are not coplanar, then the vector \(\left [ \left ( \bar{a} \times \bar{b} \right ) \times \left ( \bar{c} \times \bar{d} \right )+\left ( \bar{a} \times \bar{c} \right ) \times \left ( \bar{d} \times \bar{b} \right ) +\left ( \bar{a} \times \bar{d} \right ) \times \left ( \bar{b} \times \bar{c} \right ) \right ]\) is


Please choose your answer from the right side options

parallel to \(\bar{a}\)

parallel to \(\bar{b}\)

parallel to \(\bar{c}\)

perpendicular to \(\bar{a}\)

1520183135

\(x_1,x_2,...,x_n\) are \(n\) observations with mean \(\bar{x}\) and standard deviation \(σ\) , Match the items of \(List-I \) with those of \(List-II\)

  

\(List-I \) \(List-I I\)
(a)  \(\sum_{i=1}^{n}\left ( x_i-\bar{x} \right )\) (i) Median
(b) Variance ( \(σ^2\)) (ii) Coefficient of variation
(c) Mean deviation (iii) Zero
(d) Measure used to find the homogeneity of given two series (iv) Mean of the absolute deviations from any measure of central tendency
  (v) Mean of the squares of the deviations from mean

 


Please choose your answer from the right side options

(a) (b)  (c)  (d)

(i)  (iv)  (ii) (iii)

(a) (b) (c) (d)

(i) (iv) (iii) (ii)

(a) (b) (c) (d)

(iii) (v) (iv) (ii)

(a) (b) (c) (d)

(iii) (v) (ii)  (i)

1520183136

The variance of \(50\) observation is \(7\). If each observation is multiplied by \(6\) and then \(5\) is subtracted from it, then the variance of the new data is


Please choose your answer from the right side options

\(37 \)

\(42 \)

\(247 \)

\(252\)

1520183137

Two dice are thrown and two coins are tossed simultaneously. The probability of getting prime numbers on both the dice along with a head and a tail on the two coins is


Please choose your answer from the right side options

\(\frac{1}{8}\)

\(\frac{1}{2}\)

\(\frac{3}{16}\)

\(\frac{1}{4}\)

1520183138

\(5\) persons entered a lift cabin on the ground floor of a \(7\) floor house. Suppose that each of them independently and with equal probability can leave the cabin at any floor beginning with the first. The probability of all the \(5\) persons leaving the cabin at different floors, is


Please choose your answer from the right side options

\(\frac{360}{2401}\)

\(\frac{5}{54}\)

\(\frac{5}{18}\)

\(\frac{5!}{7!}\)

1520183139

A company produces 10,000 items per day. On a particular day 2500 items were produced on machine A, 3500 on machine B and 4000 on machine C. The probability that an item produced by the machines A, B, C to be defective is respectively 2%, 3% and 5%. If one item is selected at random from the output and is found to be defective, then the probability that it was produced by machine C, is


Please choose your answer from the right side options

\(\frac{10}{71}\)

\(\frac{16}{71}\)

\(\frac{40}{71}\)

\(\frac{21}{71}\)

1520183140

A random variable \(X\) takes the value \(1,2,3\) and \(4\) such that \(2P\left ( X=1 \right )=3P\left ( X=2 \right )=P\left ( X=3 \right )=5P\left ( X=4 \right )\) . If \( σ^2\) is the variance and \(µ\) is the mean of \(X\) then \( σ ^2+µ^2 =\)


Please choose your answer from the right side options

\(\frac{421}{61}\)

\(\frac{570}{61}\)

\(\frac{149}{61}\)

\(\frac{3480}{3721}\)

1520183141

An executive in company makes an average 5 telephone call per hour at a cost of Rs 2 per cell. The probability that in any hour the cost of the calls exceeds a sum of Rs.4 is


Please choose your answer from the right side options

\(\frac{2e^4-35}{2e^5}\)

\(\frac{2e^5-37}{2e^5}\)

\(1-\frac{37}{e^4}\)

\(1-(18.5)e^5\)

1520183142

A quadrilateral \(ABCD\) is divided by the diagonal \(AC\) in two triangles of equal areas. If \(A,B, C\) are respectively \((3,4),(-3,6),(-5,1),\) then the locus of \(D\) is


Please choose your answer from the right side options

\((x-8y-57)(x-8y+11)=0\)

\((x-8y-57)(x-8y-11)=0\)

\((3x-8y-57)(3x-8y+11)=0\)

\((3x-8y-11)(3x-8y+57)=0\)

1520183143

By rotating the coordinate axes in the positive direction about the origin by an angle \(α \), if the point \((1,2)\) is transformed to \(\left ( \frac{3\sqrt{3}-1}{2\sqrt{2}},\frac{\sqrt{3}+3}{2\sqrt{2}} \right )\) in new coordinate system then \( α =\)


Please choose your answer from the right side options

\(\frac{\pi}{3}\)

\(\frac{\pi}{6}\)

\(\frac{\pi}{9}\)

\(\frac{\pi}{12}\)

1520183144

Let a \(a\neq 0,b\neq 0,c\) be three real number and \(L(p,q)=\frac{ap+bq+c}{\sqrt{a^2+b^2}},\forall\, p,q\,\epsilon \,\mathbb{R}\),. If \(L\left ( \frac{2}{3},\frac{1}{3} \right )+L\left ( \frac{1}{3},\frac{2}{3} \right )+L(2,2)=0\) , then the line \(ax+by+c=0\) always passes through the fixed point


Please choose your answer from the right side options

\((0,1)\)

\((1,1)\)

\((2,2)\)

\((-1,-1)\)

1520183145

The incentre of the triangle formed by the straight line having \(3\) X-intercept and \(4\) as Y-intercept, together with the coordinate axes is


Please choose your answer from the right side options

\((2,2)\)

\(\left ( \frac{3}{2},\frac{3}{2} \right )\)

\((1,2)\)

\((1,1)\)

1520183146

The equation of the straight line in the normal form which is parallel to the lines \(x+2y+3=0\) and \(x+2y+8=0\) and dividing the distance between these two lines in the ratio \(1:2\) internally is,


Please choose your answer from the right side options

\(x\,cos\,\alpha+y\,sin\,\alpha=\frac{10}{\sqrt{45}},\,\alpha=tan^{-1}\sqrt{2}\)

\(x\,cos\,\alpha+y\,sin\,\alpha=\frac{14}{\sqrt{45}},\,\alpha=\pi+tan^{-1}{2}\)

\(x\,cos\,\alpha+y\,sin\,\alpha=\frac{14}{\sqrt{45}},\,\alpha=tan^{-1}{2}\)

\(x\,cos\,\alpha+y\,sin\,\alpha=\frac{10}{\sqrt{45}},\,\alpha=\pi+tan^{-1}\sqrt{2}\)

1520183147

A pair of straight lines is passing through the point \((1,1)\) . One of the lines makes an angle \( θ\) with the positive direction of \(X-\)axis and the other makes the same angle with the positive direction of Y-axis. If the equation of the pair of straight lines is \(x^2-(a+2)xy+y^2+a(x+y-1)=0,a\neq 2\) , then the value of \( θ\) is


Please choose your answer from the right side options

\(\frac{1}{2}sin^{-1}\left ( \frac{2}{a+2} \right )\)

\(\frac{1}{2}sin\left ( \frac{2}{a+2} \right )\)

\((-\infty ,-1]\cup [1,\infty )\)

\(\frac{1}{2}tan\left ( \frac{2}{a+2} \right )\)

1520183148

If the pair of lines  \(6x^2+xy-y^2=0\)  and  \(3x^2-axy-y^2=0\) , \(a>0\) have a common line, then \(a =\)


Please choose your answer from the right side options

\(\frac{1}{2}\)

\(1\)

\(2\)

\(4\)

1520183149

If the chord \(L\equiv y-mx-1=0\) of the circle \(S\equiv x^2+y^2-1=0\) touches the circle \(\,\,S_1\equiv x^2+y^2-4x+1=0\) , then the possible points for which \(\,\,L=0\) is a chord of contact of \(\,\,S=0\) are


Please choose your answer from the right side options

\(\left ( 2\pm \sqrt{6} ,0\right )\)

\(\,\,\left ( 2\pm \sqrt{6} ,1\right )\)

\(\,\,(2,0)\)

\(\,\,(\sqrt{6},1)\)

1520183150

If  \(y+c=0\)  is a tangent to the circle \(\,\,x^2+y^2-6x-2y+1=0\)  at \(\,\,(a,4)\) then


Please choose your answer from the right side options

\(ac=360\)

\(\,\,ac=-12\)

\(\,\,a+c=0\)

\(\,\,4a=c\)

1520183151

If the circles \(S\equiv x^2+y^2-14x+6y+33=0\) and \(\,\,S'\equiv x^2+y^2-a^2=0(a\,\epsilon\, N)\) have \(4\) common tangents, then the possible number of circles \(\,\,S'=0\) is


Please choose your answer from the right side options

\(1\)

\(2\)

\(0\)

infinite

1520183152

The center of the circle passing through the point \((1,0)\) and cutting the circles \(x^2+y^2-2x+4y+1=0\) and \(\,\,x^2+y^2+6x-2y+1=0 \) orthogonally is


Please choose your answer from the right side options

\(\left ( -\frac{2}{3} ,\frac{2}{3}\right ) \)

\(\left ( \frac{1}{2}, \frac{1}{2}\right ) \)

\((0,1) \)

\((0,0)\)

1520183153

The equation of the tangent at the point \((0,3)\) on the circle which cuts the circles \(x^2+y^2-2x+6y=0\)  ,  \(\,\,x^2+y^2-4x-2y+6=0\)    and\(\,\,x^2+y^2-12x+2y+3=0 \) orthogonally is


Please choose your answer from the right side options

\(y=3 \)

\(x=0 \)

\(3x+y-3=0 \)

\(x+3y-9=0\)

1520183154

If two tangents to the parabola \(y^2=8x\) meet the tangent at its vertex in \(M\) and \(N\) such that \(MN=4\) , then the locus of the point of intersection of those two tangents is


Please choose your answer from the right side options

\(y^2=8(x+3)\)

\(y^2=8(x-2)\)

\(y^2=8(x+2)\)

\(y^2=4(x+2)\)

1520183155

Three normals are drawn from the point \((c,0)\) to the curve \(y^2=x\) , If one of the normals is \(X-\)axis then the value of \(c\) for which the other two normals are perpendicular to each other is


Please choose your answer from the right side options

\(\frac{1}{4}\)

\(\frac{1}{2}\)

\(\frac{3}{4}\)

\(\frac{5}{8}\)

1520183156

If the normal drawn at one end of the latus rectum of the ellipse \(b^2x^2+a^2y^2=a^2b^2 \) with eccentricity \('e'\) passes through one end of the minor axis, then,


Please choose your answer from the right side options

\(e^4+e^2=2 \)

\(e^4-e^2=1 \)

\(e^4+e^2=1 \)

\(e^4+e=1\)

1520183157

A variable tangent to the ellipse \(\frac{x^2}{a^2}+\frac{y^2}{b^2}=1 \) makes intercept on both the axes. The locus of the middle point of the portion of the tangent between the coordinate axes is


Please choose your answer from the right side options

\(\frac{x^2}{b^2}+\frac{y^2}{a^2}=1 \)

\(\frac{a^2}{x^2}+\frac{b^2}{y^2}=1 \)

\(b^2x^2+a^2y^2=4 \)

\(\frac{a^2}{x^2}+\frac{b^2}{y^2}=4 \)

1520183158

If the eccentricity of a conic satisfies the equation \(2x^3+10x-13=0\) , then that conic is


Please choose your answer from the right side options

a circle 

a parabola

an ellipse 

a hyperbola

1520183159

Assertion (A): If \((−1,3,2)\) and \((5,3,2)\) are respectively the orthocenter and circumcentre of a triangle, then \((3,3,2) \) it its centroid.

Reason (R): Centroid of the triangle divides the line segment joining the orthocenter and the circumcentre in the ratio \(1:2\)


Please choose your answer from the right side options

(A) and (R) are true and (R) is the correct explanation to (A)

(A) and (R) are true but (R) is not the correct explanation to (A)

(A) is true, (R) is false.

(A) is false, (R) is true

1520183160

The lines whose direction cosine are given by the relations \(al+bm+cn=0\) and \(mn+nl+lm=0\) are


Please choose your answer from the right side options

perpendicular if  \(\frac{1}{a}+\frac{1}{b}+\frac{1}{c}=0\)

perpendicular if  \(\sqrt{a}+\sqrt{b}+\sqrt{c}=0\)

parallel if \(\frac{1}{a}+\frac{1}{b}+\frac{1}{c}=0\)

parallel if \(a+b+c=0\)

1520183161

If the plane passing through the points \( (1,2,3) ,( 2,3,1 )\)  and \((3,1,2)\) is \(ax+by+cz=1\) then \(\,\,a+2b+3c= \)


Please choose your answer from the right side options

\(0\)

\(1\)

\(6\)

\(18\)

1520183162

\(\lim_{x\rightarrow -\infty }\frac{3\left | x \right |-x}{\left | x \right |-2x}-\lim_{x\rightarrow 0}\frac{log(1+x^3)}{sin^3x}=\)


Please choose your answer from the right side options

\(1\)

\(\frac{1}{3}\)

\(\frac{4}{3}\)

\(0\)

1520183163

If \(f(x)=\left\{\begin{matrix} \frac{x-2}{\left | x-2 \right |}+a,& x<2\\ a+b, &x=2 \\ \frac{x-2}{\left | x-2 \right |}+b, & x>2 \end{matrix}\right.\) is continuous \(x=2\), then \(a+b=\)


Please choose your answer from the right side options

\(2\)

\(1\)

\(0\)

\(-1\)

1520183164

If \(f(x)=\left\{\begin{matrix} \frac{x^2\,log(cos\,x)}{log(1+x^2)},& x\neq 0\\ 0, &x=0 \\ \end{matrix}\right.\) then f is


Please choose your answer from the right side options

discontinuous at zero 

continuous but not differentiable at zero 

differentiable at zero 

not continuous and not differentiable at zero

1520183165

Match the items in \(List-A\) with those of the items of \(List-B\)

\(List-A\) \(List-B\)
(a) If \(y=\left | x \right |+\left | x-2 \right |\) then at \(\,\,x=2,\frac{dy}{dx}= \) \((i)\,\, 2\)
(b) If \(f(x)=\left | cos\,2x \right |,\)  then\(\,\,f'\left ( \frac{\pi}{4}+ \right )= \) \((ii) \,\,0\)
(c) If \(f(x)=sin\,\pi[x]\) where\(\,\,[ \bullet ]\) denotes the greatest integer function then\(\,\,f'(1-)= \) \((iii) \,\,−2\)
(d) If \(f(x)=log\,\left | x-1 \right |,x\neq 1\) then \(\,\,f'\left ( \frac{1}{2} \right )=\)  \((iv)\) Does not exist
  \((v) \) \(\frac{1}{2}\)

 


Please choose your answer from the right side options

(a) (b) (c) (d)

(v) (iii) (i) (ii)

(a) (b) (c)  (d)

(iv) (ii) (i)  (iii)

(a) (b) (c) (d)

(iv) (i) (ii) (iii)

(a) (b) (c) (d)

(i) (iii) (iv) (ii)

1520183166

If \(y=\frac{\left ( sin^{-1}\,x\right )^2}{2}\) , then \(\,\,(1-x^2)y_2-xy_1 \)


Please choose your answer from the right side options

\(y\)

\(2y\)

\(1\)

\(2\)

1520183167

If the relative errors in the base and radius and the height of a cone are same and equal to \(0.02\), then the percentage error in the volume of that cone is


Please choose your answer from the right side options

\(2\)

\(4\)

\(6\)

\(8\)

1520183168

The normal at a point \(\theta\) to the curve \(x=a(1+cos\,\theta),y=a\,sin\,\theta\) always passes through the fixed point


Please choose your answer from the right side options

\((0,a)\)

\( (2a,0) \)

\((a,0)\)

\((a,a)\)

1520183169

Let \(f(x)\) be continuous on\([0,6]\) and differentiable on \((0,6)\). Let \(f(0)=12\) and \(f(6)=-4\) . If \(g(x)=\frac{f(x)}{x+1}\) , then for some Lagrange’s constant \(c \in (0,6).{g}'(c)=\)


Please choose your answer from the right side options

\(-\frac{44}{3}\)

\(-\frac{22}{21}\)

\(\frac{32}{21}\)

\(-\frac{44}{21}\)

1520183170

If \((α ,β )\) and \(( γ ,δ ) \) where \( α<γ\) are the turning points of \(f(x)=2x^3-15x^2+36x-8\) then \(α−γ−β+δ=\)


Please choose your answer from the right side options

0

-2

2

1

1520183171

The height of the cylinder of the greatest volume that can be inscribed in a sphere of radius \(3\) is


Please choose your answer from the right side options

\(3\sqrt{3}\)

\(2\sqrt{3}\)

\(\sqrt{3}\)

\(\sqrt{2}\)

1520183172

\(\int \frac{dx}{\left ( e^x +e^{-x}\right )^2}=\)


Please choose your answer from the right side options

\(\frac{1}{2\left ( e^{2x} +1\right )}+c\)

\(-\frac{1}{2\left ( e^{2x} +1\right )}+c\)

\(\frac{1}{3\left ( e^{2x} +1\right )}+c\)

\(-\frac{1}{\left ( e^{2x} +1\right )}+c\)

1520183173

\(\int_{0}^{\pi/2}\frac{dx}{1+\left ( tan\,x \right )^\sqrt{2018}}=\)


Please choose your answer from the right side options

\(\pi\)

\(\frac{3\pi}{4}\)

\(\frac{\pi}{2}\)

\(\frac{\pi}{4}\)

1520183174

If \(\int \frac{x}{(x^2+1)(x-1)}dx=A\,log\left | x^2+1 \right |+B\,tan^{-1}\,x+C\,log\left | x-1 \right |+d,\) , then \(A+B+C =\)


Please choose your answer from the right side options

\(\frac{1}{4}\)

\(\frac{1}{2}\)

\(\frac{3}{4}\)

\(\frac{5}{4}\)

1520183175

\(\int_{0}^{\pi/2}\frac{cos^3\,x}{sin\,x+cos\,x}dx=\)


Please choose your answer from the right side options

\(\frac{\pi-1}{2}\)

\(\frac{\pi-1}{4}\)

\(\frac{1+\pi}{4}\)

\(\frac{\pi-3}{3}\)

1520183176

\(\int_{0}^{3}(2+x^2)dx=\)


Please choose your answer from the right side options

\(\lim_{n\rightarrow \infty }\frac{1}{n}\left [ 2n+\frac{1^2+2^2+3^2+...+(3n)^2}{n^2} \right ]\)

\(\lim_{n\rightarrow \infty }\frac{1}{n}\left [ 3n+\frac{1^2+2^2+3^2+...+6n^2}{n^2} \right ]\)

\(\lim_{n\rightarrow \infty }\frac{1}{n}\left [ 6n+\frac{1^2+2^2+3^2+...+9n^2}{n^2} \right ]\)

\(\lim_{n\rightarrow \infty }\frac{1}{n}\left [ 3n+\frac{1^2+2^2+3^2+...+3n^2}{n^2} \right ]\)

1520183177

The area enclosed (in square units) by the curve \(y=x^4-x^2\) , the \(X-\)axis and the vertical line passing through the two minimum points of the curve is


Please choose your answer from the right side options

\(\frac{48\sqrt{2}}{5}\)

\(\frac{5}{48\sqrt2}\)

\(\frac{7}{60\sqrt{2}}\)

\(\frac{7}{30 \sqrt{2}}\)

1520183178

The differential equation corresponding to the family of circles having centres on \(X-\)axis and passing through the origin is


Please choose your answer from the right side options

\(y^2+x^2+\frac{dy}{dx}=0 \)

\(y^2-x^2+\frac{dy}{dx}=0 \)

\(y^2+x^2+2xy\,\frac{dy}{dx}=0 \)

\(y^2-x^2-2xy\frac{dy}{dx}=0\)

1520183179

The general solution of the differential equation \((x^2+xy)y'=y^2 \) is


Please choose your answer from the right side options

\(e^{\frac{y}{x}} =cx \)

\(e^{-\frac{y}{x}} =cy \)

\(e^{-\frac{y}{x}} =cxy \)

\(e^{-\frac{2y}{x}} =cy\)

1520183180

At any point on a curve, the slope of the tangent is equal to the sum of abscissa and the product of ordinate and abscissa of that point. If the curve passes through \( (0,1)\), then the equation of the curve is


Please choose your answer from the right side options

\(y=2e^\frac{x^2}{2}-1 \)

\(y=2e^{x^{2}} \)

\(y=e^{-x^{2}} \)

\(y=2e^{-x^2}-1\)